Difference between revisions of "1962 AHSME Problems/Problem 14"

(Created page with "==Problem== Let <math>s</math> be the limiting sum of the geometric series <math>4- \frac83 + \frac{16}{9} - \dots</math>, as the number of terms increases without bound. Then <m...")
 
m (Solution)
 
(2 intermediate revisions by 2 users not shown)
Line 5: Line 5:
  
 
==Solution==
 
==Solution==
"Unsolved"
+
The infinite sum of a geometric series with first term <math>a</math> and common ratio <math>r</math> (<math>-1<r<1</math>) is <math>\frac{a}{1-r}</math>.
 +
Now, in this geometric series, <math>a=4</math>, and <math>r=-\frac23</math>. Plugging these into the formula, we get
 +
<math>\frac4{1-(-\frac23)}</math>, which simplifies to <math>\frac{12}5</math>, or <math>\boxed{2.4\textbf{ (B)}}</math>.
 +
 
 +
==See Also==
 +
{{AHSME 40p box|year=1962|before=Problem 13|num-a=15}}
 +
 
 +
[[Category:Introductory Algebra Problems]]
 +
{{MAA Notice}}

Latest revision as of 22:16, 3 October 2014

Problem

Let $s$ be the limiting sum of the geometric series $4- \frac83 + \frac{16}{9} - \dots$, as the number of terms increases without bound. Then $s$ equals:

$\textbf{(A)}\ \text{a number between 0 and 1}\qquad\textbf{(B)}\ 2.4\qquad\textbf{(C)}\ 2.5\qquad\textbf{(D)}\ 3.6\qquad\textbf{(E)}\ 12$

Solution

The infinite sum of a geometric series with first term $a$ and common ratio $r$ ($-1<r<1$) is $\frac{a}{1-r}$. Now, in this geometric series, $a=4$, and $r=-\frac23$. Plugging these into the formula, we get $\frac4{1-(-\frac23)}$, which simplifies to $\frac{12}5$, or $\boxed{2.4\textbf{ (B)}}$.

See Also

1962 AHSC (ProblemsAnswer KeyResources)
Preceded by
Problem 13
Followed by
Problem 15
1 2 3 4 5 6 7 8 9 10 11 12 13 14 15 16 17 18 19 20 21 22 23 24 25 26 27 28 29 30 31 32 33 34 35 36 37 38 39 40
All AHSME Problems and Solutions

The problems on this page are copyrighted by the Mathematical Association of America's American Mathematics Competitions. AMC logo.png